PrepScholar

Choose Your Test

  • Search Blogs By Category
  • College Admissions
  • AP and IB Exams
  • GPA and Coursework

How to Write a Perfect Synthesis Essay for the AP Language Exam

author image

Advanced Placement (AP)

body-pencil-sharpen-notebook-1

If you're planning to take the AP Language (or AP Lang) exam , you might already know that 55% of your overall exam score will be based on three essays. The first of the three essays you'll have to write on the AP Language exam is called the "synthesis essay." If you want to earn full points on this portion of the AP Lang Exam, you need to know what a synthesis essay is and what skills are assessed by the AP Lang synthesis essay.

In this article, we'll explain the different aspects of the AP Lang synthesis essay, including what skills you need to demonstrate in your synthesis essay response in order to achieve a good score. We'll also give you a full breakdown of a real AP Lang Synthesis Essay prompt, provide an analysis of an AP Lang synthesis essay example, and give you four tips for how to write a synthesis essay.

Let's get started by taking a closer look at how the AP Lang synthesis essay works!

Synthesis Essay AP Lang: What It Is and How It Works

The AP Lang synthesis essay is the first of three essays included in the Free Response section of the AP Lang exam.

The AP Lang synthesis essay portion of the Free Response section lasts for one hour total . This hour consists of a recommended 15 minute reading period and a 40 minute writing period. Keep in mind that these time allotments are merely recommendations, and that exam takers can parse out the allotted 60 minutes to complete the synthesis essay however they choose.

Now, here's what the structure of the AP Lang synthesis essay looks like. The exam presents six to seven sources that are organized around a specific topic (like alternative energy or eminent domain, which are both past synthesis exam topics).

Of these six to seven sources, at least two are visual , including at least one quantitative source (like a graph or pie chart, for example). The remaining four to five sources are print text-based, and each one contains approximately 500 words.

In addition to six to seven sources, the AP Lang exam provides a written prompt that consists of three paragraphs. The prompt will briefly explain the essay topic, then present a claim that students will respond to in an essay that synthesizes material from at least three of the sources provided.

Here's an example prompt provided by the College Board:

Directions : The following prompt is based on the accompanying six sources.

This question requires you to integrate a variety of sources into a coherent, well-written essay. Refer to the sources to support your position; avoid mere paraphrase or summary. Your argument should be central; the sources should support this argument .

Remember to attribute both direct and indirect citations.

Introduction

Television has been influential in United States presidential elections since the 1960's. But just what is this influence, and how has it affected who is elected? Has it made elections fairer and more accessible, or has it moved candidates from pursuing issues to pursuing image?

Read the following sources (including any introductory information) carefully. Then, in an essay that synthesizes at least three of the sources for support, take a position that defends, challenges, or qualifies the claim that television has had a positive impact on presidential elections.

Refer to the sources as Source A, Source B, etc.; titles are included for your convenience.

Source A (Campbell) Source B (Hart and Triece) Source C (Menand) Source D (Chart) Source E (Ranney) Source F (Koppel)

Like we mentioned earlier, this prompt gives you a topic — which it briefly explains — then asks you to take a position. In this case, you'll have to choose a stance on whether television has positively or negatively affected U.S. elections. You're also given six sources to evaluate and use in your response. Now that you have everything you need, now your job is to write an amazing synthesis essay.

But what does "synthesize" mean, exactly? According to the CollegeBoard, when an essay prompt asks you to synthesize, it means that you should "combine different perspectives from sources to form a support of a coherent position" in writing. In other words, a synthesis essay asks you to state your claim on a topic, then highlight the relationships between several sources that support your claim on that topic. Additionally, you'll need to cite specific evidence from your sources to prove your point.

The synthesis essay counts for six of the total points on the AP Lang exam . Students can receive 0-1 points for writing a thesis statement in the essay, 0-4 based on incorporation of evidence and commentary, and 0-1 points based on sophistication of thought and demonstrated complex understanding of the topic.

You'll be evaluated based on how effectively you do the following in your AP Lang synthesis essay:

Write a thesis that responds to the exam prompt with a defensible position

Provide specific evidence that to support all claims in your line of reasoning from at least three of the sources provided, and clearly and consistently explain how the evidence you include supports your line of reasoning

Demonstrate sophistication of thought by either crafting a thoughtful argument, situating the argument in a broader context, explaining the limitations of an argument

Make rhetorical choices that strengthen your argument and/or employ a vivid and persuasive style throughout your essay.

If your synthesis essay meets the criteria above, then there's a good chance you'll score well on this portion of the AP Lang exam!

If you're looking for even more information on scoring, the College Board has posted the AP Lang Free Response grading rubric on its website. ( You can find it here. ) We recommend taking a close look at it since it includes additional details about the synthesis essay scoring.

body-chisel-break-apart

Don't be intimidated...we're going to teach you how to break down even the hardest AP synthesis essay prompt.

Full Breakdown of a Real AP Lang Synthesis Essay Prompt

In this section, we'll teach you how to analyze and respond to a synthesis essay prompt in five easy steps, including suggested time frames for each step of the process.

Step 1: Analyze the Prompt

The very first thing to do when the clock starts running is read and analyze the prompt. To demonstrate how to do this, we'll look at the sample AP Lang synthesis essay prompt below. This prompt comes straight from the 2018 AP Lang exam:

Eminent domain is the power governments have to acquire property from private owners for public use. The rationale behind eminent domain is that governments have greater legal authority over lands within their dominion than do private owners. Eminent domain has been instituted in one way or another throughout the world for hundreds of years.

Carefully read the following six sources, including the introductory information for each source. Then synthesize material from at least three of the sources and incorporate it into a coherent, well-developed essay that defends, challenges, or qualifies the notion that eminent domain is productive and beneficial.

Your argument should be the focus of your essay. Use the sources to develop your argument and explain the reasoning for it. Avoid merely summarizing the sources. Indicate clearly which sources you are drawing from, whether through direct quotation, paraphrase, or summary. You may cite the sources as Source A, Source B, etc., or by using the descriptions in parentheses.

On first read, you might be nervous about how to answer this prompt...especially if you don't know what eminent domain is! But if you break the prompt down into chunks, you'll be able to figure out what the prompt is asking you to do in no time flat.

To get a full understanding of what this prompt wants you to do, you need to identify the most important details in this prompt, paragraph by paragraph. Here's what each paragraph is asking you to do:

  • Paragraph 1: The prompt presents and briefly explains the topic that you'll be writing your synthesis essay about. That topic is the concept of eminent domain.
  • Paragraph 2: The prompt presents a specific claim about the concept of eminent domain in this paragraph: Eminent domain is productive and beneficial. This paragraph instructs you to decide whether you want to defend, challenge, or qualify that claim in your synthesis essay , and use material from at least three of the sources provided in order to do so.
  • Paragraph 3: In the last paragraph of the prompt, the exam gives you clear instructions about how to approach writing your synthesis essay . First, make your argument the focus of the essay. Second, use material from at least three of the sources to develop and explain your argument. Third, provide commentary on the material you include, and provide proper citations when you incorporate quotations, paraphrases, or summaries from the sources provided.

So basically, you'll have to agree with, disagree with, or qualify the claim stated in the prompt, then use at least three sources substantiate your answer. Since you probably don't know much about eminent domain, you'll probably decide on your position after you read the provided sources.

To make good use of your time on the exam, you should spend around 2 minutes reading the prompt and making note of what it's asking you to do. That will leave you plenty of time to read the sources provided, which is the next step to writing a synthesis essay.

Step 2: Read the Sources Carefully

After you closely read the prompt and make note of the most important details, you need to read all of the sources provided. It's tempting to skip one or two sources to save time--but we recommend you don't do this. That's because you'll need a thorough understanding of the topic before you can accurately address the prompt!

For the sample exam prompt included above, there are six sources provided. We're not going to include all of the sources in this article, but you can view the six sources from this question on the 2018 AP Lang exam here . The sources include five print-text sources and one visual source, which is a cartoon.

As you read the sources, it's important to read quickly and carefully. Don't rush! Keep your pencil in hand to quickly mark important passages that you might want to use as evidence in your synthesis. While you're reading the sources and marking passages, you want to think about how the information you're reading influences your stance on the issue (in this case, eminent domain).

When you finish reading, take a few seconds to summarize, in a phrase or sentence, whether the source defends, challenges, or qualifies whether eminent domain is beneficial (which is the claim in the prompt) . Though it might not feel like you have time for this, it's important to give yourself these notes about each source so you know how you can use each one as evidence in your essay.

Here's what we mean: say you want to challenge the idea that eminent domain is useful. If you've jotted down notes about each source and what it's saying, it will be easier for you to pull the relevant information into your outline and your essay.

So how much time should you spend reading the provided sources? The AP Lang exam recommends taking 15 minutes to read the sources . If you spend around two of those minutes reading and breaking down the essay prompt, it makes sense to spend the remaining 13 minutes reading and annotating the sources.

If you finish reading and annotating early, you can always move on to drafting your synthesis essay. But make sure you're taking your time and reading carefully! It's better to use a little extra time reading and understanding the sources now so that you don't have to go back and re-read the sources later.

body-weightlifting-lift-strong

A strong thesis will do a lot of heavy lifting in your essay. (See what we did there?)

Step 3: Write a Strong Thesis Statement

After you've analyzed the prompt and thoroughly read the sources, the next thing you need to do in order to write a good synthesis essay is write a strong thesis statement .

The great news about writing a thesis statement for this synthesis essay is that you have all the tools you need to do it at your fingertips. All you have to do in order to write your thesis statement is decide what your stance is in relationship to the topic provided.

In the example prompt provided earlier, you're essentially given three choices for how to frame your thesis statement: you can either defend, challenge, or qualify a claim that's been provided by the prompt, that eminent domain is productive and beneficial . Here's what that means for each option:

If you choose to defend the claim, your job will be to prove that the claim is correct . In this case, you'll have to show that eminent domain is a good thing.

If you choose to challenge the claim, you'll argue that the claim is incorrect. In other words, you'll argue that eminent domain isn't productive or beneficial.

If you choose to qualify, that means you'll agree with part of the claim, but disagree with another part of the claim. For instance, you may argue that eminent domain can be a productive tool for governments, but it's not beneficial for property owners. Or maybe you argue that eminent domain is useful in certain circumstances, but not in others.

When you decide whether you want your synthesis essay to defend, challenge, or qualify that claim, you need to convey that stance clearly in your thesis statement. You want to avoid simply restating the claim provided in the prompt, summarizing the issue without making a coherent claim, or writing a thesis that doesn't respond to the prompt.

Here's an example of a thesis statement that received full points on the eminent domain synthesis essay:

Although eminent domain can be misused to benefit private interests at the expense of citizens, it is a vital tool of any government that intends to have any influence on the land it governs beyond that of written law.

This thesis statement received full points because it states a defensible position and establishes a line of reasoning on the issue of eminent domain. It states the author's position (that some parts of eminent domain are good, but others are bad), then goes on to explain why the author thinks that (it's good because it allows the government to do its job, but it's bad because the government can misuse its power.)

Because this example thesis statement states a defensible position and establishes a line of reasoning, it can be elaborated upon in the body of the essay through sub-claims, supporting evidence, and commentary. And a solid argument is key to getting a six on your synthesis essay for AP Lang!

Looking for help studying for your AP exam? Our one-on-one online AP tutoring services can help you prepare for your AP exams. Get matched with a top tutor who got a high score on the exam you're studying for!

Step 4: Create a Bare-Bones Essay Outline

Once you've got your thesis statement drafted, you have the foundation you need to develop a bare bones outline for your synthesis essay. Developing an outline might seem like it's a waste of your precious time, but if you develop your outline well, it will actually save you time when you start writing your essay.

With that in mind, we recommend spending 5 to 10 minutes outlining your synthesis essay . If you use a bare-bones outline like the one below, labeling each piece of content that you need to include in your essay draft, you should be able to develop out the most important pieces of the synthesis before you even draft the actual essay.

To help you see how this can work on test day, we've created a sample outline for you. You can even memorize this outline to help you out on test day! In the outline below, you'll find places to fill in a thesis statement, body paragraph topic sentences, evidence from the sources provided, and commentary :

  • Present the context surrounding the essay topic in a couple of sentences (this is a good place to use what you learned about the major opinions or controversies about the topic from reading your sources).
  • Write a straightforward, clear, and concise thesis statement that presents your stance on the topic
  • Topic sentence presenting first supporting point or claim
  • Evidence #1
  • Commentary on Evidence #1
  • Evidence #2 (if needed)
  • Commentary on Evidence #2 (if needed)
  • Topic sentence presenting second supporting point or claim
  • Topic sentence presenting three supporting point or claim
  • Sums up the main line of reasoning that you developed and defended throughout the essay
  • Reiterates the thesis statement

Taking the time to develop these crucial pieces of the synthesis in a bare-bones outline will give you a map for your final essay. Once you have a map, writing the essay will be much easier.

Step 5: Draft Your Essay Response

The great thing about taking a few minutes to develop an outline is that you can develop it out into your essay draft. After you take about 5 to 10 minutes to outline your synthesis essay, you can use the remaining 30 to 35 minutes to draft your essay and review it.

Since you'll outline your essay before you start drafting, writing the essay should be pretty straightforward. You'll already know how many paragraphs you're going to write, what the topic of each paragraph will be, and what quotations, paraphrases, or summaries you're going to include in each paragraph from the sources provided. You'll just have to fill in one of the most important parts of your synthesis—your commentary.

Commentaries are your explanation of why your evidence supports the argument you've outlined in your thesis. Your commentary is where you actually make your argument, which is why it's such a critical part of your synthesis essay.

When thinking about what to say in your commentary, remember one thing the AP Lang synthesis essay prompt specifies: don't just summarize the sources. Instead, as you provide commentary on the evidence you incorporate, you need to explain how that evidence supports or undermines your thesis statement . You should include commentary that offers a thoughtful or novel perspective on the evidence from your sources to develop your argument.

One very important thing to remember as you draft out your essay is to cite your sources. The AP Lang exam synthesis essay prompt indicates that you can use generic labels for the sources provided (e.g. "Source 1," "Source 2," "Source 3," etc.). The exam prompt will indicate which label corresponds with which source, so you'll need to make sure you pay attention and cite sources accurately. You can cite your sources in the sentence where you introduce a quote, summary, or paraphrase, or you can use a parenthetical citation. Citing your sources affects your score on the synthesis essay, so remembering to do this is important.

body-green-arrow-down

Keep reading for a real-life example of a great AP synthesis essay response!

Real-Life AP Synthesis Essay Example and Analysis

If you're still wondering how to write a synthesis essay, examples of real essays from past AP Lang exams can make things clearer. These real-life student AP synthesis essay responses can be great for helping you understand how to write a synthesis essay that will knock the graders' socks off .

While there are multiple essay examples online, we've chosen one to take a closer look at. We're going to give you a brief analysis of one of these example student synthesis essays from the 2019 AP Lang Exam below!

Example Synthesis Essay AP Lang Response

To get started, let's look at the official prompt for the 2019 synthesis essay:

In response to our society's increasing demand for energy, large-scale wind power has drawn attention from governments and consumers as a potential alternative to traditional materials that fuel our power grids, such as coal, oil, natural gas, water, or even newer sources such as nuclear or solar power. Yet the establishment of large-scale, commercial-grade wind farms is often the subject of controversy for a variety of reasons.

Carefully read the six sources, found on the AP English Language and Composition 2019 Exam (Question 1), including the introductory information for each source. Write an essay that synthesizes material from at least three of the sources and develops your position on the most important factors that an individual or agency should consider when deciding whether to establish a wind farm.

Source A (photo) Source B (Layton) Source C (Seltenrich) Source D (Brown) Source E (Rule) Source F (Molla)

In your response you should do the following:

  • Respond to the prompt with a thesis presents a defensible position.
  • Select and use evidence from at least 3 of the provided sources to support your line of reasoning. Indicate clearly the sources used through direct quotation, paraphrase, or summary. Sources may be cited as Source A, Source B, etc., or by using the description in parentheses.
  • Explain how the evidence supports your line of reasoning.
  • Use appropriate grammar and punctuation in communicating your argument.

Now that you know exactly what the prompt asked students to do on the 2019 AP Lang synthesis essay, here's an AP Lang synthesis essay example, written by a real student on the AP Lang exam in 2019:

[1] The situation has been known for years, and still very little is being done: alternative power is the only way to reliably power the changing world. The draw of power coming from industry and private life is overwhelming current sources of non-renewable power, and with dwindling supplies of fossil fuels, it is merely a matter of time before coal and gas fuel plants are no longer in operation. So one viable alternative is wind power. But as with all things, there are pros and cons. The main factors for power companies to consider when building wind farms are environmental boon, aesthetic, and economic factors.

[2] The environmental benefits of using wind power are well-known and proven. Wind power is, as qualified by Source B, undeniably clean and renewable. From their production requiring very little in the way of dangerous materials to their lack of fuel, besides that which occurs naturally, wind power is by far one of the least environmentally impactful sources of power available. In addition, wind power by way of gearbox and advanced blade materials, has the highest percentage of energy retention. According to Source F, wind power retains 1,164% of the energy put into the system – meaning that it increases the energy converted from fuel (wind) to electricity 10 times! No other method of electricity production is even half that efficient. The efficiency and clean nature of wind power are important to consider, especially because they contribute back to power companies economically.

[3] Economically, wind power is both a boon and a bone to electric companies and other users. For consumers, wind power is very cheap, leading to lower bills than from any other source. Consumers also get an indirect reimbursement by way of taxes (Source D). In one Texan town, McCamey, tax revenue increased 30% from a wind farm being erected in the town. This helps to finance improvements to the town. But, there is no doubt that wind power is also hurting the power companies. Although, as renewable power goes, wind is incredibly cheap, it is still significantly more expensive than fossil fuels. So, while it is helping to cut down on emissions, it costs electric companies more than traditional fossil fuel plants. While the general economic trend is positive, there are some setbacks which must be overcome before wind power can take over as truly more effective than fossil fuels.

[4] Aesthetics may be the greatest setback for power companies. Although there may be significant economic and environmental benefit to wind power, people will always fight to preserve pure, unspoiled land. Unfortunately, not much can be done to improve the visual aesthetics of the turbines. White paint is the most common choice because it "[is] associated with cleanliness." (Source E). But, this can make it stand out like a sore thumb, and make the gargantuan machines seem more out of place. The site can also not be altered because it affects generating capacity. Sound is almost worse of a concern because it interrupts personal productivity by interrupting people's sleep patterns. One thing for power companies to consider is working with turbine manufacturing to make the machines less aesthetically impactful, so as to garner greater public support.

[5] As with most things, wind power has no easy answer. It is the responsibility of the companies building them to weigh the benefits and the consequences. But, by balancing economics, efficiency, and aesthetics, power companies can create a solution which balances human impact with environmental preservation.

And that's an entire AP Lang synthesis essay example, written in response to a real AP Lang exam prompt! It's important to remember AP Lang exam synthesis essay prompts are always similarly structured and worded, and students often respond in around the same number of paragraphs as what you see in the example essay response above.

Next, let's analyze this example essay and talk about what it does effectively, where it could be improved upon, and what score past exam scorers awarded it.

To get started on an analysis of the sample synthesis essay, let's look at the scoring commentary provided by the College Board:

  • For development of thesis, the essay received 1 out of 1 possible points
  • For evidence and commentary, the essay received 4 out of 4 possible points
  • For sophistication of thought, the essay received 0 out of 1 possible points.

This means that the final score for this example essay was a 5 out of 6 possible points . Let's look more closely at the content of the example essay to figure out why it received this score breakdown.

Thesis Development

The thesis statement is one of the three main categories that is taken into consideration when you're awarded points on this portion of the exam. This sample essay received 1 out of 1 total points.

Now, here's why: the thesis statement clearly and concisely conveys a position on the topic presented in the prompt--alternative energy and wind power--and defines the most important factors that power companies should consider when deciding whether to establish a wind farm.

Evidence and Commentary

The second key category taken into consideration when synthesis exams are evaluated is incorporation of evidence and commentary. This sample received 4 out of 4 possible points for this portion of the synthesis essay. At bare minimum, this sample essay meets the requirement mentioned in the prompt that the writer incorporate evidence from at least three of the sources provided.

On top of that, the writer does a good job of connecting the incorporated evidence back to the claim made in the thesis statement through effective commentary. The commentary in this sample essay is effective because it goes beyond just summarizing what the provided sources say. Instead, it explains and analyzes the evidence presented in the selected sources and connects them back to supporting points the writer makes in each body paragraph.

Finally, the writer of the essay also received points for evidence and commentary because the writer developed and supported a consistent line of reasoning throughout the essay . This line of reasoning is summed up in the fourth paragraph in the following sentence: "One thing for power companies to consider is working with turbine manufacturing to make the machines less aesthetically impactful, so as to garner greater public support."

Because the writer did a good job consistently developing their argument and incorporating evidence, they received full marks in this category. So far, so good!

Sophistication of Thought

Now, we know that this essay received a score of 5 out of 6 total points, and the place where the writer lost a point was on the basis of sophistication of thought, for which the writer received 0 out of 1 points. That's because this sample essay makes several generalizations and vague claims where it could have instead made specific claims that support a more balanced argument.

For example, in the following sentence from the 5th paragraph of the sample essay, the writer misses the opportunity to state specific possibilities that power companies should consider for wind energy . Instead, the writer is ambiguous and non-committal, saying, "As with most things, wind power has no easy answer. It is the responsibility of the companies building them to weigh the benefits and consequences."

If the writer of this essay was interested in trying to get that 6th point on the synthesis essay response, they could consider making more specific claims. For instance, they could state the specific benefits and consequences power companies should consider when deciding whether to establish a wind farm. These could include things like environmental impacts, economic impacts, or even population density!

Despite losing one point in the last category, this example synthesis essay is a strong one. It's well-developed, thoughtfully written, and advances an argument on the exam topic using evidence and support throughout.

body-number-four-post-it-note

4 Tips for How to Write a Synthesis Essay

AP Lang is a timed exam, so you have to pick and choose what you want to focus on in the limited time you're given to write the synthesis essay. Keep reading to get our expert advice on what you should focus on during your exam.

Tip 1: Read the Prompt First

It may sound obvious, but when you're pressed for time, it's easy to get flustered. Just remember: when it comes time to write the synthesis essay, read the prompt first !

Why is it so important to read the prompt before you read the sources? Because when you're aware of what kind of question you're trying to answer, you'll be able to read the sources more strategically. The prompt will help give you a sense of what claims, points, facts, or opinions to be looking for as you read the sources.

Reading the sources without having read the prompt first is kind of like trying to drive while wearing a blindfold: you can probably do it, but it's likely not going to end well!

Tip 2: Make Notes While You Read

During the 15-minute reading period at the beginning of the synthesis essay, you'll be reading through the sources as quickly as you can. After all, you're probably anxious to start writing!

While it's definitely important to make good use of your time, it's also important to read closely enough that you understand your sources. Careful reading will allow you to identify parts of the sources that will help you support your thesis statement in your essay, too.

As you read the sources, consider marking helpful passages with a star or check mark in the margins of the exam so you know which parts of the text to quickly re-read as you form your synthesis essay. You might also consider summing up the key points or position of each source in a sentence or a few words when you finish reading each source during the reading period. Doing so will help you know where each source stands on the topic given and help you pick the three (or more!) that will bolster your synthesis argument.

Tip 3: Start With the Thesis Statement

If you don't start your synthesis essay with a strong thesis statement, it's going to be tough to write an effective synthesis essay. As soon as you finish reading and annotating the provided sources, the thing you want to do next is write a strong thesis statement.

According to the CollegeBoard grading guidelines for the AP Lang synthesis essay, a strong thesis statement will respond to the prompt— not restate or rephrase the prompt. A good thesis will take a clear, defensible position on the topic presented in the prompt and the sources.

In other words, to write a solid thesis statement to guide the rest of your synthesis essay, you need to think about your position on the topic at hand and then make a claim about the topic based on your position. This position will either be defending, challenging, or qualifying the claim made in the essay's prompt.

The defensible position that you establish in your thesis statement will guide your argument in the rest of the essay, so it's important to do this first. Once you have a strong thesis statement, you can begin outlining your essay.

Tip 4: Focus on Your Commentary

Writing thoughtful, original commentary that explains your argument and your sources is important. In fact, doing this well will earn you four points (out of a total of six)!

AP Lang provides six to seven sources for you on the exam, and you'll be expected to incorporate quotations, paraphrases, or summaries from at least three of those sources into your synthesis essay and interpret that evidence for the reader.

While incorporating evidence is very important, in order to get the extra point for "sophistication of thought" on the synthesis essay, it's important to spend more time thinking about your commentary on the evidence you choose to incorporate. The commentary is your chance to show original thinking, strong rhetorical skills, and clearly explain how the evidence you've included supports the stance you laid out in your thesis statement.

To earn the 6th possible point on the synthesis essay, make sure your commentary demonstrates a nuanced understanding of the source material, explains this nuanced understanding, and places the evidence incorporated from the sources in conversation with each other. To do this, make sure you're avoiding vague language. Be specific when you can, and always tie your commentary back to your thesis!

body-person-arrows-next

What's Next?

There's a lot more to the AP Language exam than just the synthesis essay. Be sure to check out our expert guide to the entire exam , then learn more about the tricky multiple choice section .

Is the AP Lang exam hard...or is it easy? See how it stacks up to other AP tests on our list of the hardest AP exams .

Did you know there are technically two English AP exams? You can learn more about the second English AP test, the AP Literature exam, in this article . And if you're confused about whether you should take the AP Lang or AP Lit test , we can help you make that decision, too.

Want to improve your SAT score by 160 points or your ACT score by 4 points?   We've written a guide for each test about the top 5 strategies you must be using to have a shot at improving your score. Download them for free now:

Trending Now

How to Get Into Harvard and the Ivy League

How to Get a Perfect 4.0 GPA

How to Write an Amazing College Essay

What Exactly Are Colleges Looking For?

ACT vs. SAT: Which Test Should You Take?

When should you take the SAT or ACT?

Get Your Free

PrepScholar

Find Your Target SAT Score

Free Complete Official SAT Practice Tests

How to Get a Perfect SAT Score, by an Expert Full Scorer

Score 800 on SAT Math

Score 800 on SAT Reading and Writing

How to Improve Your Low SAT Score

Score 600 on SAT Math

Score 600 on SAT Reading and Writing

Find Your Target ACT Score

Complete Official Free ACT Practice Tests

How to Get a Perfect ACT Score, by a 36 Full Scorer

Get a 36 on ACT English

Get a 36 on ACT Math

Get a 36 on ACT Reading

Get a 36 on ACT Science

How to Improve Your Low ACT Score

Get a 24 on ACT English

Get a 24 on ACT Math

Get a 24 on ACT Reading

Get a 24 on ACT Science

Stay Informed

Get the latest articles and test prep tips!

Follow us on Facebook (icon)

Ashley Sufflé Robinson has a Ph.D. in 19th Century English Literature. As a content writer for PrepScholar, Ashley is passionate about giving college-bound students the in-depth information they need to get into the school of their dreams.

Ask a Question Below

Have any questions about this article or other topics? Ask below and we'll reply!

What are your chances of acceptance?

Calculate for all schools, your chance of acceptance.

Duke University

Your chancing factors

Extracurriculars.

ap lang synthesis essay overview

How to Write the AP Lang Synthesis Essay + Example

Do you know how to improve your profile for college applications.

See how your profile ranks among thousands of other students using CollegeVine. Calculate your chances at your dream schools and learn what areas you need to improve right now — it only takes 3 minutes and it's 100% free.

Show me what areas I need to improve

What’s Covered:

What is the ap lang synthesis essay, how will ap scores affect my college chances.

AP English Language and Composition, commonly known as AP Lang, is one of the most engaging and popular AP classes offered at most high schools, with over 535,000 students taking the class . AP Lang tests your ability to analyze written pieces, synthesize information, write rhetorical essays, and create cohesive and concrete arguments. However, the class is rather challenging as only 62% of students were able to score a three or higher on the exam. 

The AP Lang exam has two sections. The first consists of 45 multiple choice questions which need to be completed in an hour. This portion counts for around 45% of your total score. These questions ask students to analyze written pieces and answer questions related to each respective passage.  All possible answer choices can be found within the text, and no prior knowledge of literature is needed to understand the passages.

The second section contains three free-response questions to be finished in under two hours and 15 minutes. This section counts for 55% of your score and includes the synthesis essay, the rhetorical essay, and the argumentative essay.

  • The synthesis essay requires you to read 6-7 sources and create an argument using at least three sources.
  • The rhetorical analysis essay requires you to describe how a piece of writing evokes specific meanings and symbolism.
  • The argumentative essay requires you to pick a perspective of a debate and create an argument based on the evidence provided.

In this post, we will take a look at the AP Lang synthesis essay and discuss tips and tricks to master this part of the exam. We will also provide an example of a well-written essay for review.  

The AP Lang synthesis essay is the first of three essays included in the Free Response section of the AP Lang exam. The exam presents 6-7 sources that are organized around a specific topic, with two of those sources purely visual, including a single quantitative source (like a graph or pie chart). The remaining 4-5 sources are text-based, containing around 500 words each. It’s recommended that students spend an hour on this essay—15 minute reading period, 40 minutes writing, and 5 minutes of spare time to check over work.

Each synthesis essay has a topic that all the sources will relate to. A prompt will explaining the topic and provide some background, although the topics are usually broad so you will probably know something related to the issue. It will also present a claim that students will respond to in an essay format using information from at least three of the provided sources. You will need to take a stance, either agreeing or disagreeing with the position provided in the claim. 

According to the CollegeBoard, they are looking for essays that “combine different perspectives from sources to form a support of a coherent position.” This means that you must state your claim on the topic and highlight relationships between several sources that support your specific position on the topic. Additionally, you’ll need to cite clear evidence from your sources to prove your point.

The synthesis essay counts for six points on the AP Lang exam. Students can receive 0-1 points for writing a thesis statement, 0-4 based on the incorporation of evidence and commentary, and 0-1 points based on the sophistication of thought and demonstration of complex understanding.

While this essay seems extremely overwhelming, considering there are a total of three free-response essays to complete, with proper time management and practiced skills, this essay is manageable and straightforward. In order to enhance the time management aspect of the test to the best of your ability, it is essential to divide the essay up into five key steps.

Step 1: Analyze the Prompt

As soon as the clock starts, carefully read and analyze what the prompt asks from you. It might be helpful to markup the text to identify the most critical details. You should only spend around 2 minutes reading the prompt so you have enough time to read all the sources and figure out your argument. Don’t feel like you need to immediately pick your stance on the claim right after reading the prompt. You should read the sources before you commit to your argument.

Step 2: Read the Sources Carefully

Although you are only required to use 3 of the 6-7 sources provides, make sure you read ALL of the sources. This will allow you to better understand the topic and make the most educated decision of which sources to use in your essay. Since there are a lot of sources to get through, you will need to read quickly and carefully.

Annotating will be your best friend during the reading period. Highlight and mark important concepts or lines from each passage that would be helpful in your essay. Your argument will probably begin forming in your head as you go through the passages, so you will save yourself a lot of time later on if you take a few seconds to write down notes in the margins. After you’ve finished reading a source, reflect on whether the source defends, challenges, or qualifies your argument.

You will have around 13 minutes to read through all the sources, but it’s very possible you will finish earlier if you are a fast reader. Take the leftover time to start developing your thesis and organizing your thoughts into an outline so you have more time to write. 

Step 3: Write a Strong Thesis Statement 

In order to write a good thesis statement, all you have to do is decide your stance on the claim provided in the prompt and give an overview of your evidence. You essentially have three choices on how to frame your thesis statement: You can defend, challenge or qualify a claim that’s been provided by the prompt. 

  • If you are defending the claim, your job will be to prove that the claim is correct .
  • If you are challenging the claim, your job will be to prove that the claim is incorrect .
  • If you choose to qualify the claim, your job will be to agree to a part of the claim and disagree with another part of the claim. 

A strong thesis statement will clearly state your stance without summarizing the issue or regurgitating the claim. The CollegeBoard is looking for a thesis statement that “states a defensible position and establishes a line of reasoning on the issue provided in the prompt.”

Step 4: Create a Minimal Essay Outline

Developing an outline might seem like a waste of time when you are up against the clock, but believe us, taking 5-10 minutes to outline your essay will be much more useful in the long run than jumping right into the essay.

Your outline should include your thesis statement and three main pieces of evidence that will constitute each body paragraph. Under each piece of evidence should be 2-3 details from the sources that you will use to back up your claim and some commentary on how that evidence proves your thesis.

Step 5: Write your Essay

Use the remaining 30-35 minutes to write your essay. This should be relatively easy if you took the time to mark up the sources and have a detailed outline.  Remember to add special consideration and emphasis to the commentary sections of the supporting arguments outlined in your thesis. These sentences are critical to the overall flow of the essay and where you will be explaining how the evidence supports or undermines the claim in the prompt.

Also, when referencing your sources, write the in-text citations as follows: “Source 1,” “Source 2,” “Source 3,” etc. Make sure to pay attention to which source is which in order to not incorrectly cite your sources. In-text citations will impact your score on the essay and are an integral part of the process.

After you finish writing, read through your essay for any grammatical errors or mistakes before you move onto the next essay.

Here are six must-have tips and tricks to get a good score on the synthesis essay:

  • Cite at least four sources , even though the minimum requirement is three. Remember not to plagiarize and cite everything you use in your arguments.
  • Make sure to develop a solid and clear thesis . Develop a stable stance for the claim and stick with it throughout the entire paper.
  • Don’t summarize the sources. The summary of the sources does not count as an argument. 
  • You don’t necessarily have to agree with the sources in order to cite them. Using a source to support a counterargument is still a good use of a source.
  • Cite the sources that you understand entirely . If you don’t, it could come back to bite you in the end. 
  • Use small quotes , do not quote entire paragraphs. Make sure the quote does not disrupt the flow or grammar of the sentence you write. 

ap lang synthesis essay overview

Discover your chances at hundreds of schools

Our free chancing engine takes into account your history, background, test scores, and extracurricular activities to show you your real chances of admission—and how to improve them.

Here is an example prompt and essay from 2019 that received 5 of the 6 total points available:

In response to our society’s increasing demand for energy, large-scale wind power has drawn attention from governments and consumers as a potential alternative to traditional materials that fuel our power grids, such as coal, oil, natural gas, water, or even newer sources such as nuclear or solar power. Yet the establishment of large-scale, commercial-grade wind farms is often the subject of controversy for a variety of reasons.

Carefully read the six sources, found on the AP English Language and Composition 2019 Exam (Question 1), including the introductory information for each source. Write an essay that synthesizes material from at least three of the sources and develops your position on the most important factors that an individual or agency should consider when deciding whether to establish a wind farm.

Source A (photo)

Source B (Layton)

Source C (Seltenrich)

Source D (Brown)

Source E (Rule)

Source F (Molla)

In your response you should do the following:

  • Respond to the prompt with a thesis presents a defensible position.
  • Select and use evidence from at least 3 of the provided sources to support your line of reasoning. Indicate clearly the sources used through direct quotation, paraphrase, or summary. Sources may be cited as Source A, Source B, etc., or by using the description in parentheses.
  • Explain how the evidence supports your line of reasoning.
  • Use appropriate grammar and punctuation in communicating your argument.

[1] The situation has been known for years, and still very little is being done: alternative power is the only way to reliably power the changing world. The draw of power coming from industry and private life is overwhelming current sources of non-renewable power, and with dwindling supplies of fossil fuels, it is merely a matter of time before coal and gas fuel plants are no longer in operation. So one viable alternative is wind power. But as with all things, there are pros and cons. The main factors for power companies to consider when building wind farms are environmental boon, aesthetic, and economic factors.

[2] The environmental benefits of using wind power are well-known and proven. Wind power is, as qualified by Source B, undeniably clean and renewable. From their production requiring very little in the way of dangerous materials to their lack of fuel, besides that which occurs naturally, wind power is by far one of the least environmentally impactful sources of power available. In addition, wind power by way of gearbox and advanced blade materials, has the highest percentage of energy retention. According to Source F, wind power retains 1,164% of the energy put into the system – meaning that it increases the energy converted from fuel (wind) to electricity 10 times! No other method of electricity production is even half that efficient. The efficiency and clean nature of wind power are important to consider, especially because they contribute back to power companies economically.

[3] Economically, wind power is both a boon and a bone to electric companies and other users. For consumers, wind power is very cheap, leading to lower bills than from any other source. Consumers also get an indirect reimbursement by way of taxes (Source D). In one Texan town, McCamey, tax revenue increased 30% from a wind farm being erected in the town. This helps to finance improvements to the town. But, there is no doubt that wind power is also hurting the power companies. Although, as renewable power goes, wind is incredibly cheap, it is still significantly more expensive than fossil fuels. So, while it is helping to cut down on emissions, it costs electric companies more than traditional fossil fuel plants. While the general economic trend is positive, there are some setbacks which must be overcome before wind power can take over as truly more effective than fossil fuels.

[4] Aesthetics may be the greatest setback for power companies. Although there may be significant economic and environmental benefit to wind power, people will always fight to preserve pure, unspoiled land. Unfortunately, not much can be done to improve the visual aesthetics of the turbines. White paint is the most common choice because it “[is] associated with cleanliness.” (Source E). But, this can make it stand out like a sore thumb, and make the gargantuan machines seem more out of place. The site can also not be altered because it affects generating capacity. Sound is almost worse of a concern because it interrupts personal productivity by interrupting people’s sleep patterns. One thing for power companies to consider is working with turbine manufacturing to make the machines less aesthetically impactful, so as to garner greater public support.

[5] As with most things, wind power has no easy answer. It is the responsibility of the companies building them to weigh the benefits and the consequences. But, by balancing economics, efficiency, and aesthetics, power companies can create a solution which balances human impact with environmental preservation.

More examples can be found here at College Board.

While AP Scores help to boost your weighted GPA, or give you the option to get college credit, AP Scores don’t have a strong effect on your admissions chances . However, colleges can still see your self-reported scores, so you might not want to automatically send scores to colleges if they are lower than a 3. That being said, admissions officers care far more about your grade in an AP class than your score on the exam.

Related CollegeVine Blog Posts

ap lang synthesis essay overview

Are you seeking one-on-one college counseling and/or essay support? Limited spots are now available. Click here to learn more.

How to Write the AP Lang Synthesis Essay with Example

September 5, 2023

If you’re highly interested in learning more about writing analysis, then chances are you enrolled in AP Lang. Essentially, AP Lang is an advanced course for high schoolers that combines interest and knowledge in English with critical thinking. In the class, students learn how to analyze and synthesize a variety of texts to construct well-reasoned arguments. If you take AP Lang, then you can opt to take the AP test at the conclusion of the school year. On the exam, students write the AP Lang synthesis essay to demonstrate their learned abilities. In this article, we’ll look at what the AP Lang synthesis essay requires and show an example to provide better understanding of what to expect on the exam.

AP Lang Exam Basics

The AP Lang exam is separated into two sections. In the first section, students have one hour to answer a series of 45 multiple-choice questions. Here, about half of the questions are based on passages students read. The other half are focused on the best revision techniques. Essentially, the answers for the latter 20-22 questions are geared toward revising mock essays.

In this article, however, we’ll focus mainly on the second part of the exam: the AP Lang synthesis essay.

In this second section, students have two hours and 15 minutes to write three essays of their own design. The three open-ended questions in this section are intended to be free-response and allow for a variety of approaches. Each question is intended to allow up to 40 minutes to complete.

For the AP Lang synthesis essay, students are presented with a scenario of the College Board’s design. The scenario will provide its own thesis statement. Usually, scenarios relate to real-world problems like environmental concerns, media, or government policies.

For each scenario, students are provided with 6-7 outside sources. These sources could be in the form of an image, visual graph, or written paragraph. For written paragraphs, the sources are usually no more than 500 words.

Students are then expected to incorporate at least 3-4 of these outside sources into their essay response. The outside sources are intended to be used as supporting evidence for the student’s chosen stance or argument. Students are able to either agree with or disagree with the thesis presented in the original scenario.

AP Lang Exam – Scoring

In the second part of the AP Lang exam, students can earn a possible 6 points on each essay. 1 point is earned for the development of a thesis. Up to 4 points can be earned for evidence and commentary. The final 1 point is earned for sophistication of thought.

AP Lang Exam – Takeaways

Ultimately, the goal of the AP Lang synthesis essay is not whether the student is “right” or “wrong” in their argument. The key is that students are able to reasonably and clearly support their argument using the provided sources as evidence .

The College Board looks for your ability to identify relationships between texts , form a coherent argument , and interpret external sources .

Synthesis Essay AP Lang Examples

If you’re not sure how the questions will look on the AP Lang synthesis essay section, we’ll provide an example. After the example, we’ll break down the strengths and weaknesses of the response. That way, you’ll have a better idea of what the College Board is looking for.

Additionally, the College Board has released previous AP Lang synthesis essay examples you can review. They even have essay questions as recent as 2022 . For further support, a scoring commentary and comments from the Chief Reader are also available to view. Additionally, there are other examples you can view from earlier years .

Note: A good strategy to study for the synthesis essay AP Lang exam is to review your rhetorical devices and literary devices . Understanding how these devices function can be essential in constructing a cohesive essay.

Synthesis Essay AP Lang Examples – Sample Question

Below is a sample question from the AP Lang synthesis essay and a response to the prompt. This question was taken directly from a 2022 exam . However, the response to the question will be originally crafted for the purpose of this newsletter. As well, all supporting evidence will be originally created and does not correspond to any previous test.

The Question

Since the early 2000s, the United States government and a number of corporations have sponsored initiatives to improve education in the STEM disciplines: science, technology, engineering, and mathematics. The emphasis on STEM subjects in elementary, secondary, and higher education reflects concerns that United States students are less proficient in these areas than are students in other countries. Additionally, there is a belief that mastery in STEM fields is now essential in order to join a highly technical and specialized workforce. However, not everyone is convinced that a STEM-focused curriculum is necessary and/or effective.

In your response you should do the following:

  • Respond to the prompt with a thesis that presents a defensible
  • Select and use evidence from at least three of the provided sources to support your line of Indicate clearly the sources used through direct quotation, paraphrase, or summary. Sources may be cited as Source A, Source B, etc., or by using the description in parentheses.
  • Explain how the evidence supports your line of
  • Use appropriate grammar and punctuation in communicating your

How to Approach the Question

Maybe your first thought upon seeing this block of text is to feel overwhelmed. But don’t panic. There are effective ways to approach the question so you will be more prepared in your response.

It’s a good strategy to first isolate the thesis . What is the main idea of the text, and what is its argument?

Try it out. Reread the prompt and see if you can identify what the statement is asking you to develop an opinion on.

Think you’ve got it? In this example, we will be focusing on whether or not a STEM-focused curriculum in K-12 education is necessary and/or effective. In short, we will be arguing either for (highlighting the benefits) or against (highlighting the pitfalls) a STEM-focused curriculum.

How do we know what this statement is asking us?

Well, the statement provides a lot of background information. For example, we receive a definition of what STEM stands for. As well, we know that since 2000, there has been a greater initiative for STEM-focused classes.

When you read the prompt for the first time, it’s a great strategy to learn how to differentiate between background and contextual information from the heart of the argument .

A good way to learn how to isolate the argument is to look for transition words. Usually, these appear near the end of the question. Words like “however” and “yet” are signals that the statement is offering a differing opinion. Typically, the statement will tell you which two positions it’s offering for argument. These opinions are usually signaled by contrasting transition words.

So, now that we know what the question is asking us, what is the best way to respond?

Synthesis Essay AP Lang Examples – Sample Answer

The following is an essay response I crafted to the above question. After reading the sample, I will break down what it does well and what areas can be improved.

A STEM-focused curriculum is not as essential to providing a meaningful K-12 education. Because the majority of high school students are not proficient in STEM-focused classes, prioritizing these classes causes harm to student’s mental health and academic performance.

As seen in Source A, 60% of high school seniors in the Midwest only scored a C average in math and science-based classes (Langston). This statistic suggests that the majority of students do not resonate with STEM classes and therefore perform poorly. Earning a low score in any class does not bode well for students’ mental health.

When looking at the primary argument in Source C, it’s clear that most high schoolers prefer creative outlets to fact-based research (Kohler). Allowing students the opportunity to be more creative and initiate conversations about coursework lets students be more active in their learning. When students can discuss the nuance in their opinions, more personal growth happens. These conversations are not always easy to have in STEM-focused classes.

As well, when looking back to Source A, it’s clear that high school students in the Midwest earned higher grades, on average, in their English and art classes (Langston). This figure suggests that students perform better in these classes because they relate more to the source material. When relating to what they learn, they perform better in class.

In conclusion, STEM-focused curriculum is not as essential in K-12 education because most high school students do not relate to their STEM classes. When students do not earn satisfactory grades in these classes, it negatively affects their future college applications and job prospects.

Synthesis Essay AP Lang Examples – Answer Breakdown

So, what does this essay response get right, and where can it be improved? Let’s start with what the response does well.

First, the response establishes its thesis right away. Usually, it’s a good idea to clearly state your argument within the first paragraph. Not only is this a good practice because a reader can easily identify your stance, but also you can refer to your thesis as you write to make you stay on track.

With your thesis, it’s also a good idea to include one to two supporting sentences with the reasons why the thesis is concluded . Like in this example, I wrote that STEM-focused classes should not be prioritized because they can negatively affect both mental health and academic performance.

Another positive aspect of this response is that it is sure to not only reference but also cites its sources . It’s important that the reader understand where your information is coming from. That way, the readers can ensure you are interpreting the sources correctly.

AP Lang Synthesis Essay (Continued)

However, when rereading the instructions, it’s clear that this response fails the basic requirement of referring to at least three sources. Always make sure to reread the instructions to ensure you meet the standard requirements for incorporating source material.

Further, this AP Lang synthesis essay does not fully support its arguments . Ideas are simply stated and are not expanded upon.

For example, I mentioned a few times that earning low grades in STEM classes leads to negative mental health for high school students. However, there is no source referenced that either confirms or denies this claim. Therefore, there is no sufficient evidence to support my argument. It relies purely on inference.

Additionally, this AP Lang synthesis essay does not arrive at a sufficient level of sophistication of thought . Basically, sophistication of thought means avoiding broad generalizations and vague claims. The more specific you can be, the better your argument will sound.

Synthesis Essay AP Lang – In Conclusion

In the end, it’s always helpful to read the prompt thoroughly before writing. As well, making notes while you read could be a good strategy to pinpoint main ideas both in the prompt and the sources. That way, you can reread the material quickly. Similarly, sketching an outline may also be helpful. In addition, you should always carefully read the instructions to ensure all guidelines are followed.

As long as you avoid broad generalizations and use enough supporting evidence for your claim, you will be on the right path!

  • High School Success

Meghan Dairaghi

With a BA in English and an MFA in Creative Writing, Meghan has served as a writing tutor at the University of Missouri St. Louis and Maryville University. Additionally, Meghan has held editorial roles at River Styx and Boulevard, and was a prose reader at Farside Review . Most recently, her work has been featured in Belle Ombre , Flypaper Lit , and Mag 20/20 , among others, and she was nominated for the Mary Troy Prize in Fiction. 

  • 2-Year Colleges
  • ADHD/LD/Autism/Executive Functioning
  • Application Strategies
  • Best Colleges by Major
  • Best Colleges by State
  • Big Picture
  • Career & Personality Assessment
  • College Essay
  • College Search/Knowledge
  • College Success
  • Costs & Financial Aid
  • Data Visualizations
  • Dental School Admissions
  • Extracurricular Activities
  • Graduate School Admissions
  • High Schools
  • Homeschool Resources
  • Law School Admissions
  • Medical School Admissions
  • Navigating the Admissions Process
  • Online Learning
  • Outdoor Adventure
  • Private High School Spotlight
  • Research Programs
  • Summer Program Spotlight
  • Summer Programs
  • Teacher Tools
  • Test Prep Provider Spotlight

“Innovative and invaluable…use this book as your college lifeline.”

— Lynn O'Shaughnessy

Nationally Recognized College Expert

College Planning in Your Inbox

Join our information-packed monthly newsletter.

How to Write the AP Lang Synthesis Essay

ap lang synthesis essay overview

AP Lang test is the logical conclusion to the introductory college English composition course. And its most important (and often difficult) part is the AP Lang synthesis essay. Despite it being the very basic layer of your future composition skills, it’s a very complicated challenge to approach unprepared. Besides, it's details may change year to year. So let’s have a look with our coursework writing services team at what your AP Lang exam 2022 might look like.

What is AP Lang?

AP Lang is a relatively lengthy test. There are several AP rubrics that a student must be well-versed in to hope to pass it. The first section includes reading and writing, while the second is slightly more freeform and includes three different types of essays.

Among those three, the most interesting and, coincidentally, oftentimes the hardest to deal with is the AP Lang synthesis essay rubric. Today will focus on it specifically to make sure you know exactly what you’re going to be facing during your test.

What Is a Synthesis Essay AP Lang?

At its core, the AP Lang synthesis essay is a pretty straightforward part of the AP Lang test. It might look pretty similar to the reading section of the exam. However, simply finding the right information isn’t enough. When writing a synthesis essay, you should not only gather the data but also distill it into your personal opinion.

This fine line may seem difficult to spot, but it is there. And it’s that small difference that can make or break your exam run. So try to follow the steps one by one and not lose focus. Writing a good synthesis essay is as easy as following the rules. If you feel this task is too difficult for you, you can leave us your ' write an essay for me ' request and we will do it for you.

AP Lang Synthesis Essay Outline

Looking through AP Lang essay examples, you might notice that the overall structure doesn’t really differ too much from your standard essay outline. You have your introduction, your body, and your conclusion. But the important thing to note is where your arguments are supposed to come from.

You’re not supposed to just go off on a rant. The task requires you to base your supporting evidence on at least three sources. And you will have to ensure your essay has solid roots. Here’s what a basic AP Lang exam synthesis essay outline should look like:

  • Introduction

Provide sufficient context for the topic you are about to cover. You can do a quick overview of prevailing opinions you have grasped while browsing through your source materials.

Write a short and compelling thesis statement. This will be your ground zero for the rest of the essay. So make sure it reflects your opinion. What is a thesis statement you can read in our special article.

  • Body Paragraphs

Dedicate at least one paragraph to every source you’re using. Start with presenting the evidence you have gathered from that source and go on to explain how it formed your opinion on the topic and why it should be considered.

Quickly go through your line of reasoning and reinforce what you have already covered. Finish up with restating your thesis as you’re supposed to logically arrive at it after all the evidence you have presented. That’s how you write a conclusion properly.

Different Forms and Types of Synthesis Essay: Explanatory vs. Argumentative Synthesis Essays

When it comes to writing a synthesis essay AP Lang, there are several types of essays you should consider. The most common ones are the AP Lang argument essay and explanatory essay. The clues as to how each of them should look are hidden within their names but let’s go over them to clear any confusion.

An explanatory essay’s goal is to go over a certain topic, discuss it in detail, and ultimately show a high level of understanding of the said topic. You don’t necessarily have to get into a heated argument with the reader trying to convince them of something. All you need to do is create an impartial overview.

On the other hand, an argumentative essay has to do with personal opinions. And while there is a time and a place for bias, it still has to be as impartial and factual as possible. When proving your point, try not to devolve into emotional arguments but stick to logic and cold truths. This will make your argument way more solid.

Synthesis Essay Structure

In the general case, you don’t really need to look for a synthesis essay AP Lang example to get a solid grasp on how its structure should look like. You can safely fall back on your high school essay writing knowledge, and you’ll be mostly safe.

What you should pay attention to is your writing style and content. A synthesis essay is identified less by its structure and more by the way you form and present your arguments to the reader. It’s when you get a specific essay type (like an argumentative essay) that you should pay attention to slight changes in format.

Argument Essay Structure

The best way to understand argumentative essay structure is to study any well-written AP Lang argument essay example. Standard AP Lang essays have very distinctive features that are very easy to spot and emulate. They follow a very rigid form and employ specific rhetorical devices that you’ll be able to pick up after you analyze them once or twice quickly.

How Many Paragraphs Should an AP Lang Synthesis Essay Be?

The number of paragraphs in an AP Lang synthesis essay can indeed make a difference. Your arguments should be concise and pointed. Spreading them out throughout many paragraphs may seem like a good idea to fill in the space. But it’s actually detrimental to your final score. You can get a basic understanding of what your score is going to be using an AP Lang score calculator.

The same goes for too few paragraphs. Don’t even try to squeeze your entire line of thought into a single body paragraph. Generally, the minimum number of sources you should address is three. Any less, and you are getting a lower score. So try to keep it somewhere in the middle. Three to five body paragraphs is an optimal number. Don’t forget to add an intro and a conclusion to it and you’re all set. A well-written essay has a clear and easily identifiable structure.

How to Write AP Lang Synthesis Essay: Guide

How to Write the AP Lang Synthesis Essay

In order to write a decent essay, all you have to do is follow these simple steps. Performing a rhetorical analysis essay example, AP Lang won’t give you insight into how it was built from the ground up. But looking at this list might.

Step 1. Read the Prompt

It may sound like a no-brainer. But it’s actually more important than you can imagine. Don’t skip right past this step. It’s very easy to misunderstand the task under stress. And if you do slip up in the beginning - the entirety of your work after that is wasted.

Step 2. Analyze the Sources Carefully

The same goes for your sources. Take your time reading them. Try to spot every smallest detail, as even a single one can help you better incorporate your evidence into the body of your essay. You can begin outlining the general points of your essay in your head at this point.

Step 3. Come Up with a Strong Thesis Statement

Your thesis statement is the baseline of your writing. Make it short and clear. Try not to overthink it too much.

Step 4. Fill in Your Essay Outline

Start filling out your outline step by step. You don’t have to go from top to bottom. If you feel like you’re struggling - skip to the next part and return to the problem paragraph later. The use of rhetorical devices AP Lang is also pretty important. So once you flesh out your essay a bit, spend some time trying to come up with the perfect wording.

Step 5. Finalize

The first finished version of your essay is a draft. Don’t be hasty to turn it in. Read over it a couple of times. Make sure everything is in order. You can switch some of the parts around or rewrite some sections if you have the time. Ideally, at this stage you should have enough time to eliminate all grammatical errors that may still be present in your essay. Polish it to perfection.

Useful Tips

Here are some useful tips that might make the writing process a bit easier for you:

  • Use either APA or Chicago style to cite your sources
  • Have a schedule to understand how much time you have for each section
  • Leave as much time as you can for editing and proofreading
  • You can never over study the source material. Spend as much time as you can reading into it
  • Don’t linger on the surface of your essay subject. Dive in and show your complex understanding of the material
  • Avoid using private life anecdotes to support your case unless the essay type specifically allows it. These don’t make for a convincing argument.
  • Use as many supporting arguments as you can but make sure they are actually solid and relevant to your thesis
  • Check with your thesis from time to time. The entirety of your text should align with it

Need help with academic deadlines?

Falling back on your deadlines? Use our term paper writing services to relieve you while you get back on your feet.

AP Lang Essay Prompts

Here are some interesting prompts. Some of them could be found in the previous iterations of the test; you may have spotted them in some of the AP Lang essay examples. Others are there to help you practice for the AP Lang exam 2022.

  • The John F. Kennedy Presidential Library and Museum, dedicated in 1979, was founded in memory of the president and contained archives pertaining to his administration. On June 24, 1985, then President Ronald Reagan joined members of the Kennedy family at a fundraising event to help the Kennedy Library Foundation create an endowment to fund and support the presidential library. The following is an excerpt from the speech Reagan gave at that event. Read the passage carefully. Write an essay that analyzes the rhetorical choices Reagan makes to achieve his purpose of paying tribute to John F. Kennedy.
  • On August 29, 2009, then-President Barack Obama delivered a eulogy at the funeral of Senator Ted Kennedy in Boston, Massachusetts. Kennedy served in the United States Senate from 1962 until his death. Obama served with him in the Senate from 2005 until Obama was elected president in 2008. The following is an excerpt from Obama’s speech. Read the passage carefully. Write an essay that analyzes the rhetorical choices Obama makes to achieve his purpose of praising and memorializing Kennedy.
  • On April 9, 1964, Claudia “Lady Bird” Johnson, who was at the time the First Lady of the United States, gave the following speech at the first-anniversary luncheon of the Eleanor Roosevelt Memorial Foundation. The foundation is a nonprofit division of the Franklin D. Roosevelt Presidential Library dedicated to the works of former First Lady Eleanor Roosevelt, who passed away in 1962. Read the passage carefully. Write an essay that analyzes the rhetorical choices Johnson makes to achieve her purpose of paying tribute to Eleanor Roosevelt.

In your response, you should do the following:

• Respond to the prompt with a thesis that analyzes the writer’s rhetorical choices.

• Select and use evidence to support your line of reasoning.

• Explain how the evidence supports your line of reasoning.

• Demonstrate an understanding of the rhetorical situation.

• Use appropriate grammar and punctuation in communicating your argument.

AP Lang Essay Example

Here is a decent if a bit shortened, AP Lang rhetorical analysis essay example you can use for reference.

Literature to Prepare for AP Lang

How to Write the AP Lang Synthesis Essay

And here is a list of some great AP Lang books that will help you prepare for the exam. Not all of them are immediately useful, but most will help you enhance your writing and analytical abilities to get a better score in the end.

  • The Odyssey
  • Don Quixote
  • A Midsummer Night's Dream
  • Pride and Prejudice
  • Wuthering Heights
  • Oliver Twist
  • Crime and Punishment
  • Adventures of Huckleberry Finn

If you have thoughts of "who could do my paper for me," do not forget that you can contact us. Or, if you have a finished paper and you need to make edits to it, leave us a ' rewrite my essay ' request and we will do it as soon as possible.

Daniel Parker

Daniel Parker

is a seasoned educational writer focusing on scholarship guidance, research papers, and various forms of academic essays including reflective and narrative essays. His expertise also extends to detailed case studies. A scholar with a background in English Literature and Education, Daniel’s work on EssayPro blog aims to support students in achieving academic excellence and securing scholarships. His hobbies include reading classic literature and participating in academic forums.

ap lang synthesis essay overview

is an expert in nursing and healthcare, with a strong background in history, law, and literature. Holding advanced degrees in nursing and public health, his analytical approach and comprehensive knowledge help students navigate complex topics. On EssayPro blog, Adam provides insightful articles on everything from historical analysis to the intricacies of healthcare policies. In his downtime, he enjoys historical documentaries and volunteering at local clinics.

How to Write a Personal Narrative

Calculate for all schools

Your chance of acceptance, your chancing factors, extracurriculars, writing a synthesis essay for ap lang.

I have to write a synthesis essay this week for my AP Lang class, but I'm not 100% sure what exactly that entails. Can someone explain what the process is like and maybe share some tips on how to write a strong synthesis essay?

Sure, a synthesis essay is a type of essay that requires you to use multiple sources to create an argument. In an AP Lang synthesis essay, you'll typically be provided with the sources and will need to analyze them, identify the main ideas, and then connect those ideas to your central argument or thesis.

Here's a step-by-step breakdown of how to write a strong synthesis essay for AP Lang:

1. Understand the prompt : Carefully read the prompt and make sure you understand what's being asked. Identify the main issue or topic you'll be addressing.

2. Develop a thesis statement : Your thesis statement should make a clear and specific claim about the topic. It should be an arguable point that you'll support using evidence from the sources provided.

3. Read and analyze the sources : Read through each source, taking notes on key ideas, evidence, and arguments related to your thesis. Think about how each source contributes to your argument and look for connections between sources.

4. Outline your essay : Create an outline that organizes your ideas and evidence into a logical sequence. Typically, a synthesis essay will have an introduction, body paragraphs, and conclusion. Each body paragraph should focus on a specific point related to your thesis and include evidence from multiple sources to support the point.

5. Write the introduction : Begin with a hook to engage the reader and introduce the topic. Then, give some brief background information about the issue before presenting your thesis statement.

6. Write the body paragraphs : In each body paragraph, start with a topic sentence that introduces the main point of the paragraph. Then, provide evidence and analysis from the sources to support the point, making sure to cite your sources according to the citation style required by your instructor. Try to incorporate evidence from multiple sources and emphasize the connections between them.

7. Address counterarguments : It's important to address any opposing views or counterarguments to show that you've considered different perspectives. You can either refute counterarguments or concede some validity while still maintaining your overall argument.

8. Write the conclusion : Summarize your main points and restate your thesis statement in a new way. You can also suggest further implications of your argument or propose a call to action. Avoid introducing new information in the conclusion.

Some additional tips for writing a strong synthesis essay:

- Make sure your analysis connects effectively to your thesis statement.

- Clearly explain how the evidence from the sources supports your argument.

- Use appropriate transitions to guide your reader through the essay and help them understand the connections between ideas.

- Edit and revise your essay for clarity, grammar, and punctuation.

Good luck with your AP Lang synthesis essay!

About CollegeVine’s Expert FAQ

CollegeVine’s Q&A seeks to offer informed perspectives on commonly asked admissions questions. Every answer is refined and validated by our team of admissions experts to ensure it resonates with trusted knowledge in the field.

  • Arizona State University (ASU)
  • Boston University
  • Dartmouth College
  • Georgetown University
  • Harvard University
  • Massachusetts Institute of Technology​
  • Our Contacts
  • Our Gallery
  • Our Pricing
  • Our Services
  • Purdue University Indianapolis (PU Indy)
  • Sample Page
  • universities
  • University of Florida
  • University of Michigan, Ann Arbor

How to Write the AP Lang Synthesis Essay with Example

How to Write the AP Lang Synthesis Essay with Example

Are you a high school student preparing for the AP Language and Composition exam? Or perhaps you are a teacher looking to help your students with the skills to ace the synthesis essay? Either way, you’ve landed in the right place. This blog will serve as your comprehensive guide to mastering this challenging yet rewarding component of the AP Language exam.

We’ll dig into what a synthesis essay entails and its structure, and we’ll furnish you with actionable strategies to approach the task with confidence. We’ll also provide insights into selecting and integrating sources effectively, constructing a compelling argument, and polishing your writing to perfection.

Table of Contents

Overview of AP Language and Composition

AP English Language and Composition , widely known as AP Lang, is a popular and engaging Advanced Placement course taken by over half a million high school students each year. The course is designed to hone essential skills such as analyzing written works, synthesizing information, constructing rhetorical essays, and writing compelling arguments. While the course presents a rigorous challenge, with just over 60% of students achieving a passing score of three or higher on the AP exam, the rewards of mastering these skills are significant.

The AP Lang exam is a comprehensive assessment consisting of two distinct sections. The first section, a one-hour multiple-choice segment, assesses your ability to analyze written passages and answer questions based solely on the provided text. This section comprises approximately 45% of the total exam score. The second section is a two-hour and fifteen-minute free-response segment. It evaluates your writing skills through three distinct essays. This section accounts for the remaining 55% of the exam score.

The three essays within the free-response section target specific writing skills. The synthesis essay challenges you to develop an argument by incorporating information from multiple provided sources. The rhetorical analysis essay requires you to dissect how an author uses language to convey meaning and achieve specific effects. Finally, the argumentative essay prompts you to take a stance on a debatable issue and construct a persuasive argument based on evidence.

What is the AP Lang Synthesis Essay?

The AP Language and Composition exam’s first free-response task is the synthesis essay. It is a one-hour exercise during which you read six to seven sources on a specific topic and compose a well-developed essay. These sources include a mix of print texts, approximately 500 words each, and visual elements like graphs or charts. You are advised to allocate 15 minutes to reading and analyzing these sources, followed by 40 minutes for writing and 5 minutes for review, but the time distribution can be adjusted as needed.

The synthesis essay prompt comprises three paragraphs: a brief introduction to the topic, a claim about the topic, and instructions for the essay. The claim is often broad and open to interpretation, requiring you to take a stance—either agreeing or disagreeing—and support your position by synthesizing information from at least three of the provided sources.

According to the College Board, a successful synthesis essay should “ combine different perspectives from sources to form a support of a coherent position. ” This means you must clearly state your claim, establish connections between sources to reinforce your argument, and provide specific evidence to validate your points.

The synthesis essay contributes six points to the overall AP Lang exam score. A holistic rubric evaluates the essay based on the thesis statement (0–1 point), evidence and commentary (0–4 points), and sophistication of thought and complexity of understanding (0–1 point).

Here’s an example prompt and essay provided by the College Board :


Urban rewilding is an effort to restore natural ecological processes and habitats in city environments. Many cities around the world have embraced rewilding as part of larger movements to promote ecological conservation and environmentally friendly design. Now, a movement to promote urban rewilding is beginning to take shape in the United States as well.



Refer to the sources as Source A, Source B, etc.; titles are included for your convenience.
Source A (infographic from Fastnacht)
Source B (Jepson and Schepers policy brief)
Source C (NRPA article)
Source D (Garland article)
Source E (graph from McDonald et al.)
Source F (Chatterton book excerpt)

In your response, you should do the following:
1. Respond to the prompt with a thesis that presents a defensible position.
2. Select and use evidence from at least three of the provided sources to support your line of reasoning.
3. Indicate clearly the sources used through direct quotation, paraphrase, or summary. Sources may be cited as Source A, Source B, etc., or by using the description in parentheses.
4. Explain how the evidence supports your line of reasoning.
5. Use appropriate grammar and punctuation when communicating your argument.

Rewilding is a term that not many people have heard of or even pay attention to. That doesn’t mean it’s not important, either. Rewilding is a good thing for the planet; it’s good for plants and the environment. The world needs to start caring, and children, especially, are the future.

Rewilding is good for our environment and for the future of preserving our world. In source C, “If people don’t spend any time outside, why are they going to care about their local places, let alone the national parks in the distance?” Going outside isn’t just good for the planet; it is also good for yourself. Nature isn’t really welcome in big cities, but reintroducing new plants can make it feel like it is welcome. Kids need to start caring about nature and not just about phones and video games. It gives you a different way to see our planet and care about what happens to it.

In addition, rewilding is valuable for our society to learn as a whole. In source B, “Rewilding is exciting, engaging, and challenging; it is promoting debate and deliberation on what is natural and the natures we collectively wish to conserve and shape.” It’s important for kids to understand, and a challenge can be what a lot of children need. Also in source A, “More than 70% of projected extinctions of plants and animals would be counteracted by restoring only 30% of priority areas.” That can be such a good thing, and that’s why rewilding, especially for our country, is important. If we don’t, we could lose 70% of plants and animals, which would send the ecosystem into whack.

Overall, rewilding should be focused on more; we have a lot to lose. Putting in the time and effort in our cities and urban settings is what we need to do. If you don’t care now, start caring. Kids especially need to focus.

Read also: Write an ap lang argument essay

How to Write a Synthesis Essay for the AP Language Exam

Step 1: analyze the prompt.

Begin by carefully reading and analyzing the prompt. Underline or highlight key terms to identify the central question and your task. Remember that you don’t need to decide your stance immediately; understanding the prompt is the priority here.

Step 2: Read and annotate the sources

Although you’ll only use three sources in your essay, read them all. This provides a broader understanding of the topic and helps you choose the most relevant evidence. As you read, actively annotate by highlighting key points, noting connections, and jotting down potential arguments.

After each source, briefly assess whether it supports, opposes, or nuances your emerging thesis. If you finish reading early, use the remaining time to start outlining your essay.

Step 3: Write a strong thesis statement

Your thesis statement should clearly state your position on the prompt’s claim. You can choose to defend the claim (argue it’s correct), challenge it (argue it’s incorrect), or qualify it (agree with some aspects and disagree with others). A strong thesis avoids summarizing the issue or restating the prompt; it establishes a clear line of reasoning.

Step 4: Outline your essay

Though it may seem counter intuitive when time is limited, outlining is essential. Your outline should include your thesis statement, three main points (one for each body paragraph), and the supporting evidence you’ll use from the sources. Briefly note how this evidence connects back to your thesis.

Step 5: Write your essay

With your annotated sources and outline in hand, writing your essay should be smoother. Begin with a focus on providing insightful commentary that explains how your evidence supports or refutes the prompt’s claim.

When referencing sources, use simple in-text citations like “Source 1,” “Source 2,” etc. Be sure to double-check your citations for accuracy. Before moving on, quickly proofread your essay for any errors.

Read also: How Long Should Your College Essay Be?

AP Lang Synthesis Essay Score Evaluation

The AP Language Synthesis Essay accounts for six points of the total exam score. Your essay will be evaluated on several key components. Primarily, a clear and defensible thesis statement that directly responds to the exam prompt can earn you up to one point. The majority of your score (up to four points) depends on how well you incorporate evidence from at least three sources and explain how that evidence supports your reasoning. Each piece of evidence should be explicitly linked to your argument, demonstrating a clear and consistent line of thought.

To earn the final point, your essay must show sophistication of thought. This can be achieved by writing a nuanced argument that acknowledges the complexities and tensions within the sources, situating your argument within a broader context to reveal its implications, or explaining the limitations of your or others’ arguments. Additionally, employing effective rhetorical devices and maintaining a vivid and persuasive writing style can further strengthen your essay.

Read also: Personal Statement Essay Examples

5 Tips to Ace the Synthesis Essay for the AP Language Exam

1. understand the prompt.

Begin by meticulously analyzing the prompt. Identify the central issue being discussed and the specific task you’re asked to perform (argue, evaluate, analyze, etc.). Underline key terms and phrases to ensure you fully grasp the expectations.

2. Engage actively with the sources

Don’t just skim through the sources; actively read and annotate the provided sources. Identify the main idea and supporting evidence in each. Note the source’s perspective and any potential biases. Highlight quotes or data you might use in your essay. Aim to understand how the sources relate to each other and the prompt.

3. Write a nuanced thesis

Your thesis should be a clear, concise statement of your position on the issue presented in the prompt. It should be specific and incorporate the nuances you’ve gleaned from the sources. Avoid merely restating the prompt; instead, offer an insightful perspective that you’ll support with evidence throughout your essay.

4. Construct a cohesive argument

Your essay should be a well-structured argument, not a mere summary of the sources. Each body paragraph should focus on a single point that supports your thesis. Use evidence from the sources to back up your claims, and provide your analysis and interpretation of that evidence. Connect your paragraphs with clear transitions to create a logical flow.

5. Leave time for revision

After writing your essay, take a few minutes to review it carefully. Check for grammatical errors, awkward phrasing, and clarity issues. Ensure that your argument is well-developed and your evidence is effectively integrated. A polished essay shows your command of language and strengthens your overall argument.

ap lang synthesis essay overview

From the Desk of Yocket

Writing a good AP Language synthesis essay requires a balanced approach of critical thinking, careful analysis, and persuasive writing. You should begin by thoroughly understanding the prompt and identifying the central issue and the required task. Then, dig into the provided sources, extracting key points, perspectives, and evidence that relate to your developing stance.

A strong thesis is the backbone of your essay. It should clearly state your position on the issue and provide a roadmap for the reader, outlining the key points you’ll explore. As you bring together evidence from multiple sources, remember to provide insightful commentary, explaining how each piece of evidence bolsters your argument. Try to avoid simply dropping quotes or paraphrasing; instead, analyze the significance of each piece, showing a nuanced understanding of the issue and the sources.

You should conclude your essay by revisiting your thesis and summarizing your key arguments. You can also offer a thoughtful extension, such as suggesting implications for your argument, addressing potential counterarguments, or proposing future directions for research. Throughout your essay, prioritize clarity, coherence, and sophistication in your language and structure. This will show your ability to analyze complex texts and synthesize information into a compelling argument. Remember to maintain a strong connection with your audience, ensuring your writing on Yocket remains engaging and relevant.

Frequently Asked Questions

What is a synthesis essay on the ap language exam.

A synthesis essay requires you to develop a position on a given topic by incorporating and citing evidence from multiple sources. You’ll need to evaluate, select, and synthesize information from these sources to create a cohesive argument.

How many sources are typically provided for the synthesis essay?

The AP Language exam usually provides 6–7 sources for the synthesis essay, including texts and visual elements like graphs or charts.

What is the time allotted for writing the synthesis essay?

The entire free-response section of the AP Language exam, which includes the synthesis essay, rhetorical analysis, and argumentative essay, is 2 hours and 15 minutes. You may budget roughly 40 minutes to read the sources and plan your essay, leaving 40 minutes to write.

How is the synthesis essay scored?

The synthesis essay is scored on a 0–9 scale, with 9 being the highest. Points are awarded for a clear thesis, effective use of evidence and commentary, sophisticated analysis, and overall coherence.

Do I have to agree with the sources to use in my synthesis essay?

No, you can use sources to support a counterargument or provide alternative perspectives. The key is to engage with the sources critically and use them to build your argument.

How should I cite sources in my synthesis essay?

You can use parenthetical citations (author’s last name or source letter) to indicate where you’ve used information from the sources. It’s essential to avoid plagiarism by accurately attributing all borrowed ideas and language.

What are some common mistakes to avoid in the synthesis essay?

Try to avoid merely summarizing the sources without adding your analysis. Ensure your thesis clearly states your position and is supported by evidence throughout the essay. You should refrain from relying too heavily on one source and aim for a balanced incorporation of multiple perspectives.

  • No Comments
  • July 15, 2024

Leave a Reply Cancel reply

Your email address will not be published. Required fields are marked *

Save my name, email, and website in this browser for the next time I comment.

Get in touch with us

Are you sure you want to logout?

Study abroad.

bannerAd

Steps to Draft AP Lang Synthesis Essay

The synthesis essay AP lang is part of the AP language and composition exam . It is one of the courses in the juniors’ curriculum, making it compulsory for them to appear for the exam. Judging the argumentation skills with the ability to establish a line of reasoning, answering the exam requires students to follow a pattern. Easily cracked with the practice, here is how you should go on to learn to write the perfect answer. 

What Does the AP Language and Composition Exam Look Like?

The AP Language and Composition Exam is taken in the duration of 3 hours and 15 minutes and comprises two sections. 

parallel

  • The first section is multiple choice questions where the reading and writing abilities of the students are assessed through a total of 45 questions. The section holds 45% weightage and must be completed in an hour. 
  • The second section is free-response questions, where there are three questions holding 55% weightage. The time offered here is 2 hours and 45 minutes, where 15 minutes are allotted to the reading period. There are three essay questions here, with 40 minutes allotted to each. The first question is the synthesis AP lang essay , the second is the rhetorical analysis, and the third is an argument. 

How is the AP Lang Synthesis Essay Question Asked?

The first free-response question puts forward a specific topic and six to seven sources of information. The students are offered a variety of information where two are visuals, and at least one is the quantitative source. The remaining sources are excerpts from the text, and each of these sources will be around 500 words. The candidates are expected to synthesise their essays and reflect their viewpoints based on and backed by at least three provided sources. 

What Are the Factors or Criteria of the Assessment of the Synthesis Essay?

The synthesis essay is assessed by the invigilator considering the following points: 

parallel

  • The presence of a thesis statement in response to the prompt could start a series of argument
  • Utilisation of at least three sources to support the evidence. The sources must be clearly cited or indicated in parentheses
  • Ability to relate the evidence and thesis 
  • Capability to exhibit an understanding of the rhetorical situation 
  • Usage of proper grammar and its rules in the essay 

How To Write a Synthesis Essay AP Lang?

This is the prime question when dealing with the synthesis essay AP Lang . Let’s begin by breaking it down into smaller and more understandable components: 

Step 1: Prompt Analysis

Since you have only a completely new question and sources in hand, the first step will be to analyse the available information. Re-read the prompt at least twice or thrice and identify and note down the prime fact each prompt is conveying. For instance, the first para will be an introduction making you familiar with the topic and the second will offer insights and help you build a view or take a specific side. The third paragraph can be instructions on what is expected in the essay. 

parallel

Step 2: Choose the Source

For proper choice usage, you must know what each source encompasses. It leads to the recommendation of going through each source. Reading the source is recommended to ensure you know which sources back your side and which don’t. Summarise this information on each source with a word about which side they are on. Glancing over the sources will also aid in deciding the utilisation of sources in the topic and is efficient compared to randomly choosing the sources without reading. It needs to be done during the reading time and is aimed at saving the essay writing time. 

Step 3: Begin with the Thesis Statement

You need to begin by conveying your side or stand and must establish a line of reasoning for scoring good marks. It should be strictly related to the topic provided and must not wander about the general situations. The thesis statement needs to be clearly mentioned and should not be a copy of the complete or part of the prompt. The reader must understand at the beginning what the essay will be about or what they should expect, along with the reason for your thoughts in your write-up. 

parallel

Step 4: Structure Your Essay

An essay is supposed to be sequential. Structure with an outline is the only method to do it. This will not go on your fair copy but will be a guide on how you will approach writing the essay. You are expected to divide the essay into an introduction, body, and conclusion. The introduction will carry a brief context of the topic and sources, along with communication about your stand or side on the topic. 

The body can be divided into three paragraphs. Each paragraph will begin with supporting or opposing information, followed by the evidence and your thoughts on the same. The conclusion will be an overview or summarisation of the reasoning presented in your essay and a final line proving your point. 

parallel

Step 5: Fill the Outline With the Essay

You have the structure of what exactly you will be covering in the essay. You know how it has to go, and now the only thing left is writing the essay. Fill up your structure like the blanks. You will be communicating and reflecting your mind and thoughts to the invigilator with the essay. Ensure to depict how the evidence takes or opposes the side you are on. You can get help by incorporating quotes from the source. Overall, you will be explaining the source and clearly indicating the sources used within the parenthesis or by mention in the essay, as it is the criteria determining your marks. 

What Are the Do’s and Don’ts While Writing a Synthesis Essay AP Lang Exam?

Here are some important do’s and don’ts to follow: 

parallel

  • Don’t limit your source usage to minimum requirements. Use at least four of them.
  • Don’t end up plagiarizing by summarisation or copying the question part. 
  • Do use your freedom to agree or disagree. 
  • Do go through all the sources for better choices. 
  • Do use quotes to enhance readability 
  • Do cite the sources used 
  • Do write the thesis statement 
  • Do write the notes when going through the topic and sources 

Conclusion 

The AP lang synthesis question is a part of the second section carrying more weightage in the exam. Being one of the three free-response questions, the candidates are expected to put forward their perspective and stand while tracking back their reasoning to the sources. Going through the complete prompt and all the sources tends to assist in scoring. Further, writing assistance is available on creating a structure to fill it up with your essay. 

Frequently Asked Questions

What is the maximum possible score in the synthesis part.

The synthesis part is of 6 points, where a maximum of 1 mark is awarded on thesis development, a maximum of 4 is awarded on evidence and commentary, and a maximum of 1 mark is awarded on the sophistication of thought.

parallel

Can I score well in the AP Language and Composition exam?

Yes, you can score well. However, it necessarily requires practice, a broad way of thinking, and properly deciding the flow of ideas in the essay.

How many questions are there for reading and writing?

There are 23 to 25 questions for reading and 20 to 22 questions for writing part in section I.

AP Lang Synthesis Essay

Relevant Articles

AP Lang Rhetorical Analysis Essay

AP Lang Rhetorical Analysis Essay

AP language exams are held for numerous subjects. Students have …

AP Lang Rhetorical Analysis Essay Read More »

AP Calculus AB Exam

Strategies for Success: A Complete Overview of the AP Calculus AB Exam

The AP Calculus AB Exam serves as an important element …

Strategies for Success: A Complete Overview of the AP Calculus AB Exam Read More »

Digital AP Exams

Preparing for Success: A Guide to the Features of Digital AP Exams

Digital  Advanced Placement (AP) exams bring about a huge change …

Preparing for Success: A Guide to the Features of Digital AP Exams Read More »

card img

With Turito Study Abroad

card img

Get an Expert Advice from Turito

card img

With Turito CAP.

card img

With Turito Coding.

card img

With Turito RoboNinja

card img

1-on-1 tutoring for the undivided attention

All Subjects

Synthesis Overview - Slides

february 25, 2020

Brandon Wu

Learn what synthesis is, how to get a perfect score on the synthesis essay, and read through some samples to explore what students did well.

Fiveable

Stay Connected

© 2024 fiveable inc. all rights reserved., ap® and sat® are trademarks registered by the college board, which is not affiliated with, and does not endorse this website..

AngieSite-01.png

How Angie Kratzer’s AP English Language Resources Fit Together

ap lang synthesis essay overview

“This is overwhelming.”

“There’s so much here.”

“Where do I start?”

These three reviews of my mega AP English Language & Composition bundle made me pivot. The 50 resources make sense to ME, to MY classroom, to MY experience as a 22-year veteran teacher. In my mind, a teacher could take all the lessons, filter, pace, and order in a way that makes sense for her or his students. But what if that teacher has Lang for the first time, couldn’t make it to a summer institute, and is the only person within a 30 mile radius teaching the course? If that’s the case, that teacher is staring at a stack of 50 library books with blank covers.

Let’s put some jackets on those books. Here’s what my largest AP English Language & Composition bundle contains:

✿✿✿✿✿ Overview Resources ✿✿✿✿✿                                     

180 Days of AP English Language: A Flexible Pacing Guide                           

90 Days of AP English Language: A Flexible Pacing Guide                                                                                                                              

I’m a linear thinker, and I chunk the course by the free-response questions. I have paced this system with two guides, one for a year-long schedule and one for block. The pacing guides include hyperlinked resources so that you know which unit fits where.

AP English Language Skills Alignment Chart  

Before diving into the free response questions, get the big picture of the skills taught within each lesson plan. All skills (as listed in the current Course and Exam Description ) are spelled out in the left vertical column, and the three writing units top columns of skills included in each lesson plan.

✿✿✿✿✿ Rhetorical Analysis Resources ✿✿✿✿✿     

Tone Vocabulary Builder                       

Tone Categories Poster Pack

The foundation of rhetorical analysis is a tone vocabulary.  This tone word study will equip students to craft meaty thesis statements. Included are ten weeks’ worth of quizzes and anchor posters.

Rhetorical Analysis for Every Student                                                                  

5 Rhetorical Analysis Bell Ringers

This 27-lesson unit thoroughly walks students through two methods of organization: device-by-device and organizational. Weave in the five bell ringers as students demonstrate proficiency.

Rhetorical Analysis Review Pages 

Rhetorical Analysis Comprehensive Assessment

These two resources would work well at the end of the initial rhetorical analysis unit or as review before the May exam. The comprehensive assessment is editable, so you can make it your own, but it’s heavy on terminology. If you use it, do so with low stakes. Read more about my approach to rhetorical analysis.

7 Rhetorical Modes                                                                                                   

Rhetorical Mode Project The bundle includes three different ways to address rhetorical modes. Students need to learn both to analyze and to write in them. “7 Rhetorical Modes” offers handouts, suggested models, prompts, and rubrics. “Rhetorical Mode Project” guides students through the production of those elements. Within the thorough unit on The Things They Carried , the rhetorical modes are addressed as well. A teacher should do one of the three. Read more about teaching rhetorical modes.

Want to test drive some of these resources? Grab these six free lesson plans on teaching tone.

✿✿✿✿✿ argumentation and persuasion resources✿✿✿✿✿, ✿✿✿✿✿ synthesis ✿✿✿✿✿, ✿✿✿✿✿ multiple choice exam resources ✿✿✿✿✿, ✿✿✿✿✿ supplemental resources ✿✿✿✿✿, ✿✿✿✿✿ literature-specific resources ✿✿✿✿✿, ✿✿✿✿✿ supplemental freebies ✿✿✿✿✿, ✿✿✿✿✿✿✿✿✿✿✿✿✿✿✿✿✿✿✿✿.

AP language tone

I’m a recovering high school English teacher and curriculum specialist with a passion for helping teachers leave school at school. I create engaging, rigorous curriculum resources for secondary ELA professionals, and I facilitate workshops to help those teachers implement the materials effectively.

  • AP Language Exam
  • Argumentation & Persuasion
  • Grammar & Usage
  • Reading Instruction
  • Rhetorical Analysis
  • Teacher Tips & Best Practices
  • The Research Process
  • Uncategorized
  • Writing Instruction

Copyright © 2023  Angie Kratzer Site Design by Laine Sutherland Designs

Privacy Overview

CookieDurationDescription
cookielawinfo-checkbox-analytics11 monthsThis cookie is set by GDPR Cookie Consent plugin. The cookie is used to store the user consent for the cookies in the category "Analytics".
cookielawinfo-checkbox-functional11 monthsThe cookie is set by GDPR cookie consent to record the user consent for the cookies in the category "Functional".
cookielawinfo-checkbox-necessary11 monthsThis cookie is set by GDPR Cookie Consent plugin. The cookies is used to store the user consent for the cookies in the category "Necessary".
cookielawinfo-checkbox-others11 monthsThis cookie is set by GDPR Cookie Consent plugin. The cookie is used to store the user consent for the cookies in the category "Other.
cookielawinfo-checkbox-performance11 monthsThis cookie is set by GDPR Cookie Consent plugin. The cookie is used to store the user consent for the cookies in the category "Performance".
viewed_cookie_policy11 monthsThe cookie is set by the GDPR Cookie Consent plugin and is used to store whether or not user has consented to the use of cookies. It does not store any personal data.

IMAGES

  1. How to write a good synthesis essay ap lang

    ap lang synthesis essay overview

  2. 💄 Synthesis essay template. How to Write the AP Lang Synthesis Essay

    ap lang synthesis essay overview

  3. AmStud

    ap lang synthesis essay overview

  4. AP Lang Synthesis Essay Notes by Emily Mendez on Prezi

    ap lang synthesis essay overview

  5. Sample ap lang synthesis essay prompt in 2021

    ap lang synthesis essay overview

  6. synthesis essay on ap lang exam

    ap lang synthesis essay overview

COMMENTS

  1. Synthesis Essay Materials

    The two synthesis essay questions below are examples of the question type that has been one of the three free-response questions on the AP English Language and Composition Exam as of the May 2007 exam. The synthesis question asks students to synthesize information from a variety of sources to inform their own discussion of a topic. Students are given a 15-minute reading period to accommodate ...

  2. How to Write a Perfect Synthesis Essay for the AP Language Test

    Worried about the AP Lang synthesis essay? Check out our complete guide to writing a synthesis essay for AP Language.

  3. How to Write the AP Lang Synthesis Essay + Example

    AP English Language and Composition, commonly known as AP Lang, is one of the most engaging and popular AP classes offered at most high schools, with over 535,000 students taking the class. AP Lang tests your ability to analyze written pieces, synthesize information, write rhetorical essays, and create cohesive and concrete arguments.

  4. How to Write the AP Lang Synthesis Essay with Example

    On the exam, students write the AP Lang synthesis essay to demonstrate their learned abilities. In this article, we'll look at what the AP Lang synthesis essay requires and show an example to provide better understanding of what to expect on the exam.

  5. How to Write the AP Lang Synthesis Essay + Essay Template

    Wondering what the AP Lang exam 2024 covers and how to ace it? Dive into our guide for all the essential details, from exam structure to effective study tips.

  6. PDF AP® ENGLISH LANGUAGE AND COMPOSITION

    Overview As its label "Synthesis" suggests, Question 1 was intended to test students' abilities to combine and coordinate several academic literacy skills in concert.

  7. Writing a synthesis essay for AP Lang

    Sure, a synthesis essay is a type of essay that requires you to use multiple sources to create an argument. In an AP Lang synthesis essay, you'll typically be provided with the sources and will need to analyze them, identify the main ideas, and then connect those ideas to your central argument or thesis.

  8. AP Lang

    Overview of the Synthesis Question Section II of the AP English Language and Composition exam includes three free-response questions that you must answer in 2 hours and 15 minutes.

  9. How to Write the AP Lang Synthesis Essay with Example

    Learn essential tips for how to write AP Lang Synthesis Essay. Explore effective techniques for thesis development, argument synthesis, and impactful conclusion writing.

  10. PDF Guide to Writing the AP English Language Synthesis Essay

    Definition of "Synthesis" "For the purposes of scoring, synthesis refers to combining the sources and the writer's position to form a cohesive, supported argument and accurately citing all sources." "a cohesive, supported argument" - in other words, an argument unified around a thesis, and you support that thesis with adequate data.

  11. AP English Language and Composition

    Exam Overview Exam questions assess the course concepts and skills outlined in the course framework. For more information, download the AP English Language and Composition Course and Exam Description (.pdf) (CED).

  12. Cracking the AP Lang Synthesis Essay: Tips and Strategies

    Navigate the AP Lang and Composition exam with confidence. Learn effective strategies for crafting a standout synthesis essay and achieving success.

  13. PDF ELA Virtual Learning AP Lang and Comp: Synthesis Essay

    A synthesis is a written discussion incorporating support from several sources of difering views. This type of assignment requires that you examine a variety of sources and identify their relationship to your thesis. This is one essay of the three on the AP Lang exam.

  14. PDF AP® ENGLISH LANGUAGE AND COMPOSITION

    The essay earned an 8 for its effective argument, appropriate synthesis of sources, and consistent ability to control a wide range of the elements of effective writing, as particularly evidenced in its coherence and transitions.

  15. 2024 AP English Language and Composition Exam Guide

    This year, all AP exams will cover all units and essay types. The 2024 AP English Language and Composition exam format will be: Section I: Multiple Choice - 45% of your score- - 45 questions in 1 hour. Section II: Free Response Section - 55% of your score- - 2 hours and 15 minutes for:- 1. synthesis essay. - 1.

  16. PDF Synthesizing an Argument

    Arguably, this should be the easiest of the five canons because it is a set pattern, yet structuring the essay trips up more than its fair share of students. Fortunately for you, the Advanced Placement English Language and Composition exam's synthesis question invents the evidence for you.

  17. PDF ENGLISH LANGUAGE AND COMPOSITION

    AP English Language and Composition Question 1: Synthesis (2019) Sample Student Responses. 1. The student responses in this packet were selected from the 2019 Reading and have been rescored using the new rubrics for 2020. Commentaries for each sample are provided in a separate document.

  18. You NEED to Do THIS with Your Synthesis Topic Sentences

    This video explores how anyone can write the best topic sentences that will eventually lead to a high-scoring synthesis essay on the AP® Lang exam.💀 *GRAB T...

  19. AP Lang Synthesis Overview

    🏽 Exam Skills study guides written by former AP English Lang students to review Exam Skills with detailed explanations and practice questions.

  20. PDF AP® English Language and Composition

    Overview Students responding to this question were expected to read six sources on the topic of STEM education and then write an essay that synthesized material from at least three of the sources and developed their position on the value, if any, of initiatives to improve STEM education and increase the number of students in the STEM disciplines.

  21. PDF Microsoft Word

    The following prompt is based on the accompanying six sources. This question requires you to integrate a variety of sources into a coherent, well-written essay. Refer to the sources to support your position; avoid mere paraphrase or summary. Your argument should be central; the sources should support this argument.

  22. AP Lang Synthesis Flashcards

    Study with Quizlet and memorize flashcards containing terms like synthesis, bias, what is central to your synthesis essay? and more.

  23. How Angie Kratzer's AP English Language Resources Fit Together

    Angie Kratzer's 50-resource bundle of AP English Language & Composition resources fits together to create a complete course.